Finding the Image of a Vector under a Linear Transformation

Click For Summary
The discussion revolves around finding the image of a vector under a given linear transformation L: R^3 -> R^3, defined by its action on the basis vectors i, j, and k. The user is uncertain about how to approach the problem, considering whether to construct a matrix from the transformations of the basis vectors and apply it to the vector [2 -1 3]. There is confusion regarding the expected output, as similar problems in the text yield single number answers, while this problem should result in a vector. Participants suggest that the user should express the vector as a linear combination of the basis vectors and apply the transformation accordingly. Clarification is sought on the similar problem to identify any differences that may affect the solution.
superdave
Messages
149
Reaction score
3

Homework Statement



Let L: R^3 -> R^3 be a linear transformation such that

L(i) = [1 2 -1], L(j) = [1 0 2] and L(k) = [1 1 3].

Find L([ 2 -1 3)].

All the numbers in [ ] should be vertical, but I don't know how to set that up.

Homework Equations





The Attempt at a Solution



I'm not sure how to even approach this. I've tried looking at examples in the text and they aren't clear.

I would think that you make a matrix L with the three columns i j k as above. and x = [2 -1 3] and just calculate Lx=b to find b.

But a similar problem in the text has a single number answer. I would've guessed that it gives a vector.
 
Physics news on Phys.org
You know how the transformation acts on the basis {i, j, k}. Let v be any vector, then v = αi + βj + γk. Then L(v) = αL(i) + βL(j) + γL(k).

The vectors L(i), L(j) and L(k) are the columns of the matrix representation of the operator L.
 
superdave said:
But a similar problem in the text has a single number answer. I would've guessed that it gives a vector.

You should get a vector here. Maybe you should post the similar problem and we can help you figure out what the difference is?
 
Question: A clock's minute hand has length 4 and its hour hand has length 3. What is the distance between the tips at the moment when it is increasing most rapidly?(Putnam Exam Question) Answer: Making assumption that both the hands moves at constant angular velocities, the answer is ## \sqrt{7} .## But don't you think this assumption is somewhat doubtful and wrong?

Similar threads

  • · Replies 8 ·
Replies
8
Views
1K
  • · Replies 1 ·
Replies
1
Views
1K
  • · Replies 7 ·
Replies
7
Views
2K
Replies
15
Views
2K
  • · Replies 16 ·
Replies
16
Views
2K
Replies
10
Views
2K
  • · Replies 3 ·
Replies
3
Views
2K
  • · Replies 4 ·
Replies
4
Views
3K
  • · Replies 8 ·
Replies
8
Views
2K
  • · Replies 10 ·
Replies
10
Views
3K